Difference between revisions of "2023 AMC 12B Problems/Problem 20"

(Created page with "==Solution== Denote by <math>A_i</math> the position after the <math>i</math>th jump. Thus, to fall into the region centered at <math>A_0</math> and with radius 1, <math>\ang...")
 
(Solution)
Line 5: Line 5:
  
 
Therefore, the probability is
 
Therefore, the probability is
<math></math>
+
<cmath>
 
\[
 
\[
 
\frac{2 \cdot 2 \arcsin \frac{1}{4}}{2 \pi}
 
\frac{2 \cdot 2 \arcsin \frac{1}{4}}{2 \pi}
= \boxed{\textbf{(E) <math>\frac{2 \arcsin \frac{1}{4}}{\pi}</math>}}.
+
= \boxed{\textbf{(E) } \frac{2 \arcsin \frac{1}{4}}{\pi}}.
 
\]
 
\]
<math></math>
+
</cmath>
  
 
~Steven Chen (Professor Chen Education Palace, www.professorchenedu.com)
 
~Steven Chen (Professor Chen Education Palace, www.professorchenedu.com)

Revision as of 18:38, 15 November 2023

Solution

Denote by $A_i$ the position after the $i$th jump. Thus, to fall into the region centered at $A_0$ and with radius 1, $\angle A_2 A_1 A_0 < 2 \arcsin \frac{1/2}{2} = 2 \arcsin \frac{1}{4}$.

Therefore, the probability is \[ \frac{2 \cdot 2 \arcsin \frac{1}{4}}{2 \pi} = \boxed{\textbf{(E) } \frac{2 \arcsin \frac{1}{4}}{\pi}}. \]

~Steven Chen (Professor Chen Education Palace, www.professorchenedu.com)